Supporting Hyperplanes to the intersection of half-spaces












0












$begingroup$


Let $H(a,r)$ be a hyperplane through point $a$ with normal vector $r$, and let $H(a,r)^{+}$ be the corresponding nonnegative halfspace ,and $H(a,r)^{++}$ the positive halfspace. I have a set $Omega$ of these hyperplanes, not necessarily finite.
Consider $A equiv bigcap_Omega H(a,r)^{+}$. I know it is certainly possible that there are $H(a,r) in Omega$ that are not supporting to $A$.



But is it possible to have a point $x$ on the boundary of $A$ that is not supported by any $H(a,r) in Omega$?



I suspect not. My thinking is as follows. A point $x$ on the boundary of $A$ must lie in $H(a,r)^{+}$ for every $H(a,r)in Omega$ by construction. If $x in H(a,r)^{++}$ for all $H(a,r) in Omega$, then it cannot be on the boundary. Therefore, it must lie on at least one $H(a,r)$, and this one would be supporting.










share|cite|improve this question









$endgroup$

















    0












    $begingroup$


    Let $H(a,r)$ be a hyperplane through point $a$ with normal vector $r$, and let $H(a,r)^{+}$ be the corresponding nonnegative halfspace ,and $H(a,r)^{++}$ the positive halfspace. I have a set $Omega$ of these hyperplanes, not necessarily finite.
    Consider $A equiv bigcap_Omega H(a,r)^{+}$. I know it is certainly possible that there are $H(a,r) in Omega$ that are not supporting to $A$.



    But is it possible to have a point $x$ on the boundary of $A$ that is not supported by any $H(a,r) in Omega$?



    I suspect not. My thinking is as follows. A point $x$ on the boundary of $A$ must lie in $H(a,r)^{+}$ for every $H(a,r)in Omega$ by construction. If $x in H(a,r)^{++}$ for all $H(a,r) in Omega$, then it cannot be on the boundary. Therefore, it must lie on at least one $H(a,r)$, and this one would be supporting.










    share|cite|improve this question









    $endgroup$















      0












      0








      0





      $begingroup$


      Let $H(a,r)$ be a hyperplane through point $a$ with normal vector $r$, and let $H(a,r)^{+}$ be the corresponding nonnegative halfspace ,and $H(a,r)^{++}$ the positive halfspace. I have a set $Omega$ of these hyperplanes, not necessarily finite.
      Consider $A equiv bigcap_Omega H(a,r)^{+}$. I know it is certainly possible that there are $H(a,r) in Omega$ that are not supporting to $A$.



      But is it possible to have a point $x$ on the boundary of $A$ that is not supported by any $H(a,r) in Omega$?



      I suspect not. My thinking is as follows. A point $x$ on the boundary of $A$ must lie in $H(a,r)^{+}$ for every $H(a,r)in Omega$ by construction. If $x in H(a,r)^{++}$ for all $H(a,r) in Omega$, then it cannot be on the boundary. Therefore, it must lie on at least one $H(a,r)$, and this one would be supporting.










      share|cite|improve this question









      $endgroup$




      Let $H(a,r)$ be a hyperplane through point $a$ with normal vector $r$, and let $H(a,r)^{+}$ be the corresponding nonnegative halfspace ,and $H(a,r)^{++}$ the positive halfspace. I have a set $Omega$ of these hyperplanes, not necessarily finite.
      Consider $A equiv bigcap_Omega H(a,r)^{+}$. I know it is certainly possible that there are $H(a,r) in Omega$ that are not supporting to $A$.



      But is it possible to have a point $x$ on the boundary of $A$ that is not supported by any $H(a,r) in Omega$?



      I suspect not. My thinking is as follows. A point $x$ on the boundary of $A$ must lie in $H(a,r)^{+}$ for every $H(a,r)in Omega$ by construction. If $x in H(a,r)^{++}$ for all $H(a,r) in Omega$, then it cannot be on the boundary. Therefore, it must lie on at least one $H(a,r)$, and this one would be supporting.







      convex-analysis






      share|cite|improve this question













      share|cite|improve this question











      share|cite|improve this question




      share|cite|improve this question










      asked Nov 29 '18 at 16:06









      Mathew KnudsonMathew Knudson

      84




      84






















          0






          active

          oldest

          votes











          Your Answer





          StackExchange.ifUsing("editor", function () {
          return StackExchange.using("mathjaxEditing", function () {
          StackExchange.MarkdownEditor.creationCallbacks.add(function (editor, postfix) {
          StackExchange.mathjaxEditing.prepareWmdForMathJax(editor, postfix, [["$", "$"], ["\\(","\\)"]]);
          });
          });
          }, "mathjax-editing");

          StackExchange.ready(function() {
          var channelOptions = {
          tags: "".split(" "),
          id: "69"
          };
          initTagRenderer("".split(" "), "".split(" "), channelOptions);

          StackExchange.using("externalEditor", function() {
          // Have to fire editor after snippets, if snippets enabled
          if (StackExchange.settings.snippets.snippetsEnabled) {
          StackExchange.using("snippets", function() {
          createEditor();
          });
          }
          else {
          createEditor();
          }
          });

          function createEditor() {
          StackExchange.prepareEditor({
          heartbeatType: 'answer',
          autoActivateHeartbeat: false,
          convertImagesToLinks: true,
          noModals: true,
          showLowRepImageUploadWarning: true,
          reputationToPostImages: 10,
          bindNavPrevention: true,
          postfix: "",
          imageUploader: {
          brandingHtml: "Powered by u003ca class="icon-imgur-white" href="https://imgur.com/"u003eu003c/au003e",
          contentPolicyHtml: "User contributions licensed under u003ca href="https://creativecommons.org/licenses/by-sa/3.0/"u003ecc by-sa 3.0 with attribution requiredu003c/au003e u003ca href="https://stackoverflow.com/legal/content-policy"u003e(content policy)u003c/au003e",
          allowUrls: true
          },
          noCode: true, onDemand: true,
          discardSelector: ".discard-answer"
          ,immediatelyShowMarkdownHelp:true
          });


          }
          });














          draft saved

          draft discarded


















          StackExchange.ready(
          function () {
          StackExchange.openid.initPostLogin('.new-post-login', 'https%3a%2f%2fmath.stackexchange.com%2fquestions%2f3018836%2fsupporting-hyperplanes-to-the-intersection-of-half-spaces%23new-answer', 'question_page');
          }
          );

          Post as a guest















          Required, but never shown

























          0






          active

          oldest

          votes








          0






          active

          oldest

          votes









          active

          oldest

          votes






          active

          oldest

          votes
















          draft saved

          draft discarded




















































          Thanks for contributing an answer to Mathematics Stack Exchange!


          • Please be sure to answer the question. Provide details and share your research!

          But avoid



          • Asking for help, clarification, or responding to other answers.

          • Making statements based on opinion; back them up with references or personal experience.


          Use MathJax to format equations. MathJax reference.


          To learn more, see our tips on writing great answers.




          draft saved


          draft discarded














          StackExchange.ready(
          function () {
          StackExchange.openid.initPostLogin('.new-post-login', 'https%3a%2f%2fmath.stackexchange.com%2fquestions%2f3018836%2fsupporting-hyperplanes-to-the-intersection-of-half-spaces%23new-answer', 'question_page');
          }
          );

          Post as a guest















          Required, but never shown





















































          Required, but never shown














          Required, but never shown












          Required, but never shown







          Required, but never shown

































          Required, but never shown














          Required, but never shown












          Required, but never shown







          Required, but never shown







          Popular posts from this blog

          Plaza Victoria

          In PowerPoint, is there a keyboard shortcut for bulleted / numbered list?

          How to put 3 figures in Latex with 2 figures side by side and 1 below these side by side images but in...